Representación bajo la cual se transforman las matrices de Pauli

En la teoría cuántica de campos de Peskin y Schroeder , hay una identidad de matrices de Pauli que está conectada a la identidad de Fierz (ecuación 3.77)

(3.77) ( σ m ) α β ( σ m ) γ d = 2 ϵ α γ ϵ β d .
El autor explica que

Uno puede entender la identidad al notar que los índices α , γ transformar en la representación de Lorentz de Ψ L , mientras β , d transformarse en la representación separada de Ψ R , y la cantidad total debe ser un invariante de Lorentz.

como se puede ver α , γ y β , d transformar en diferentes representaciones?

Respuestas (4)

Las matrices de Pauli son tensores invariantes que acoplan espinores diestros y zurdos. Estos espinores se transforman en diferentes representaciones del grupo de Lorentz (como mencionaste) y, por lo tanto, generalmente se denotan con diferentes índices. Esto es trivial de ver en la notación de dos componentes, sin embargo, si no está familiarizado con esta notación, esto también se puede ver en un Lagrangiano de cuatro componentes:

ψ ¯ γ m ψ = ψ γ 0 γ m ψ = ( ψ R ψ L ) ( 0 σ m σ ¯ m 0 ) ( ψ L ψ R ) = ψ R σ m ψ R + ψ L σ ¯ m ψ L .
Entonces se puede demostrar que ψ L ( ψ R ) se transforma en un espinor diestro (zurdo). Claramente un σ m el campo luego conecta un ψ R campo con un ψ L campo. Podemos escribir estas contracciones de manera más explícita al denotar los índices de representación para zurdos con índices griegos y los índices de representación para diestros con índices griegos punteados:
ψ L α ˙ ( σ m ) α α α ˙ ψ L α

Nota: es posible que sienta la tentación de pensar en ψ R y ψ L no como campos separados, sino como campos con proyectores que actúan sobre ellos. Esto hace que todo este tema sea muy confuso y le insto a que se sienta cómodo pensando en términos de campos de dos componentes como los objetos fundamentales que forman los fermiones.

Gracias por la respuesta. Tengo la misma pregunta. Sin embargo, realmente no entendí su respuesta, ¿podría dar algunas explicaciones más específicas en términos de lenguaje simple? Entiendo que ψ ¯ γ m ψ es un vector de Lorentz, ψ es una ruleta de Lorentz que se transforma de acuerdo con una representación reducible del grupo (debido a algunos requisitos de la física). Cuando muestras eso ψ ¯ γ m ψ puede escribirse como una suma de dos términos, pero no implica (?) necesariamente que cada término sea también un vector de Lorentz.
Entonces, ¿qué significa " σ m campo conecta un campo de Weyl derecho con un campo de Weyl izquierdo"? En otras palabras, con solo mirar un término, como lo hizo, ¿cuál es el significado de la afirmación α y α ˙ pertenece a otra representación? Por último, ¿por qué la identidad de Fierz puede entenderse (derivada de una constante) en términos de estos argumentos? Disculpen mi ignorancia y muchas gracias de antemano!
¿Es correcta esta primera ecuación? Creo que debería ser ψ ¯ R σ m ψ R + ψ ¯ L σ ¯ m ψ L , por lo que no puedo encontrar lo que significa "Claramente un σ m campo conecta un ψ ¯ R campo con un ψ ¯ L campo".

Tuve la misma pregunta, y el enlace proporcionado por Qmechanic parece estar basado en una sólida comprensión de la teoría de grupos. Me preguntaba si uno puede simplemente entender la transformación de los índices para esta pregunta específica basándose solo en el libro de texto mientras usa una cantidad mínima de conocimiento/argumentos de la teoría de grupos. Después de consultar a mi colega Alberto, aquí está la respuesta que obtuve siguiendo este criterio.

La identidad (Eq.(3.77))) dice

( σ m ) α β ( σ m ) γ d = 2 ϵ α γ ϵ β d   .

Empezamos por el lado derecho de la igualdad. En primer lugar, podemos mostrar que el símbolo antisimétrico ϵ α γ es Lorentz invariante si ambos índices se transforman como espinores de Weyl zurdos

Ψ L tu L Ψ L = Exp ( i θ σ 2 β σ 2 ) Ψ L     .

La declaración anterior es equivalente a la siguiente identidad

ϵ α γ Exp ( i θ σ 2 β σ 2 ) α α Exp ( i θ σ 2 β σ 2 ) γ γ ϵ α γ = ϵ α γ
o
Exp ( i θ σ 2 β σ 2 ) ( 0 1 1 0 ) Exp ( i θ σ 2 β σ 2 ) T = ( 0 1 1 0 )

La identidad anterior puede demostrarse sin mucha dificultad observando ( 0 1 1 0 ) = i σ 2 , σ = σ y la transversal de la ecuación (3.38), de modo que

Exp ( i θ σ 2 β σ 2 ) σ 2 Exp ( i θ σ 2 β σ 2 ) T = Exp ( i θ σ 2 β σ 2 ) σ 2 [ norte 1 norte ! ( i θ σ 2 β σ 2 ) norte ] T = Exp ( i θ σ 2 β σ 2 ) Exp ( + i θ σ 2 + β σ 2 ) T σ 2 = Exp ( i θ σ 2 β σ 2 ) Exp ( + i θ σ 2 + β σ 2 ) σ 2 = Exp ( i θ σ 2 β σ 2 ) Exp ( + i θ σ 2 + β σ 2 ) σ 2 = σ 2

Un argumento muy similar muestra que ϵ α γ también es invariante si ambos índices se transforman como espinores de Weyl diestros . Entonces, uno puede optar por mirar el lado derecho de la identidad como un tensor invariante donde α , γ se transforma en la representación de Lorentz de Ψ L , mientras β , d se transforma en la representación separada de Lorentz de Ψ R , como se indica en el libro de texto.

Ahora nos movemos al lado izquierdo de la identidad. Es más difícil matemáticamente (pero aún factible) mostrar (mientras que una conjetura inteligente también apunta fuertemente) que ( σ m ) α β es también un tensor invariante de Lorentz cuando m se transforma como un vector de Lorentz definido por la ecuación (3.19), α se transforma en un espinor zurdo y β se transforma en un espinor diestro. para que cuando m se contrae en el lado izquierdo de la identidad, los parámetros libres restantes se transforman exactamente de la misma manera que los del lado derecho. Esto lleva inmediatamente a la conclusión de que la identidad es correcta hasta un número constante, que luego se puede fijar evaluando solo un término (en lugar de todos). 2 4 = dieciséis de ellos).

Por lo general, los argumentos anteriores se dan en términos del lenguaje de la teoría de grupos de una manera más elegante, y una buena referencia es la Teoría cuántica de campos de Srednicki (ver el texto entre la ecuación (34.18) y la ecuación (35.20)).

Es conveniente utilizar la notación punteada. El álgebra de Lorentz es s o ( 1 , 3 ) es isomorfo a s tu ( 2 ) L × s tu ( 2 ) R . Denotamos fundamental s tu ( 2 ) L índices por α , β , etc y fundamentales s tu ( 2 ) R índices por α ˙ , β ˙ , etc. Nótese que para representaciones unitarias de s o ( 1 , 3 ) , la conjugación compleja intercambia representaciones L y R y, por lo tanto, también intercambia índices con y sin punto.

Hay varios tensores invariantes de Lorentz de interés, en particular, d β α , d β ˙ α ˙ , ε α β , ε α ˙ β ˙ y las matrices de Pauli ( σ m ) α β ˙ Para ser absolutamente claro, lo que quiero decir es que cuando se intercalan entre espinores, se transforman de la forma indicada por sus índices (ver la respuesta de Jeff a esta pregunta). Os dejo como ejercicio de tarea probar lo que acabo de decir.

Con todos los índices explícitos, cualquier ecuación debe conservar la estructura del índice. Por ejemplo, la cantidad de interés para usted es

( σ m ) α β ˙ ( σ m ) γ d ˙
Esta es la invariante de Lorentz y tiene índices libres. α , β ˙ , γ d ˙ . La única cantidad invariante de Lorentz que se puede construir a partir de estos índices es ε α γ ε β ˙ d ˙ . Así, debemos tener
( σ m ) α β ˙ ( σ m ) γ d ˙ = λ ε α γ ε β ˙ d ˙
Lo que queda entonces es arreglar la constante λ . Para hacer esto, puede establecer β ˙ = γ y α = d ˙ y suma sobre índices repetidos (al hacer esto se rompe la invariancia de Lorentz, pero no nos importa en este punto). Entonces tenemos la ecuación matricial
tr ( σ m σ m ) = λ tr ( ε 2 ) = tr ( I 2 ) + tr ( σ σ ) = λ tr ( I 2 ) λ = 2 .
De este modo,
( σ m ) α β ˙ ( σ m ) γ d ˙ = 2 ε α γ ε β ˙ d ˙
Esto hace coincidir su ecuación con un signo que depende de la convención. He usado la convención que ε 12 = ε 1 ˙ 2 ˙ = 1 .

@Pahar Lo único que no entiendo es por qué la única cantidad invariante de Lorentz posible es ε α γ ε β ˙ d ˙ ?
Hay una diferencia de signo con tu prueba y la de Peskin. ¿Se debe a diferentes opciones métricas?
@SamapanBhadury - (1) ¿qué más podría ser? Ya he escrito todas las cantidades invariantes de Lorentz para ti. Lo que queda es construir algo a partir de esos pero con la estructura de índice correcta. (2) sí, eso suena bien. mi convención métrica es (-+++)
por qué no d α β d γ d ? ¿Por qué no podemos construir con tal d -funciones?
@SamapanBhadury - d tiene un índice superior y un índice inferior. El LHS tiene los cuatro índices bajados.
quieres decir d β α ¿no existe?
@SamapanBhadury No, no. quiero decir que d β α es un tensor invariante de Lorentz, pero d α β NO lo es y solo podemos usar tensores invariantes de Lorentz ya que LHS es un tensor invariante de Lorentz.
¿Puede usted amablemente explicar esta declaración o dirigirme a una referencia?
@SamapanBhadury: consulte el comentario anterior.

Otra forma de ver esto es usar las leyes de transformación para la γ m matrices. Tenga en cuenta que podemos escribir

Λ 1 / 2 = ( Λ 1 / 2 L 0 0 Λ 1 / 2 R ) ,
dónde Λ 1 / 2 es la matriz que realiza una transformación de Lorentz en un espinor de Dirac. Es fácil confirmar que esto es cierto porque Λ 1 / 2 está hecho de combinaciones de la identidad y las matrices diagonales de bloque S m v (asumiendo que estamos usando la representación quiral, como en Peskin y Schroesder). Es fácil ver eso Λ 1 / 2 L es el operador que realiza una transformación de Lorentz en un espinor zurdo, y Λ 1 / 2 R hace lo mismo para un espinor diestro. Para dar un ejemplo concreto, para transformaciones infinitesimales, la ecuación 3.37 dice que
Λ 1 / 2 L = 1 i θ σ / 2 β σ / 2 , y  Λ 1 / 2 R = 1 i θ σ / 2 + β σ / 2.

Ahora, podemos usar las leyes de transformación de la γ m matrices:

( Λ 1 / 2 L 1 0 0 Λ 1 / 2 R 1 ) ( 0 σ m σ ¯ m 0 ) ( Λ 1 / 2 L 0 0 Λ 1 / 2 R )
= Λ 1 / 2 1 γ m Λ 1 / 2 = Λ m v γ v = ( 0 Λ m v σ v Λ m v σ ¯ v 0 ) .
Realizando la multiplicación de matrices, y centrándonos en el bloque superior derecho, obtenemos
Λ 1 / 2 L 1 σ m Λ 1 / 2 R = Λ m v σ v .